LSAT and Law School Admissions Forum

Get expert LSAT preparation and law school admissions advice from PowerScore Test Preparation.

User avatar
 Dave Killoran
PowerScore Staff
  • PowerScore Staff
  • Posts: 5853
  • Joined: Mar 25, 2011
|
#88076
Complete Question Explanation
(The complete setup for this game can be found here: lsat/viewtopic.php?f=166&t=8547)

The correct answer choice is (B).

The condition in the question stem establishes that F is ranked third. According to our discussion of the sequence, R is the only other team that can be ranked first, and so R must be first:

G3-Q14-d1.png

The second ranked team can be difficult to deduce, but the sequence helps determine who is available:

G is not available because from the first rule F is more productive than G.
H is not available because from the fourth rule S is more productive than H.
T is not available because from the first rule F is more productive than G, and from the last rule G is more productive than T.
Thus, only S can be second, and G, H, and T must fill in the final three spaces while remaining in accordance with the last rule:

G3-Q14-d2.png

Answer choice (A) is incorrect because S must rank second.

Answer choice (B) is possible, and is therefore correct.

Answer choice (C) is incorrect because S must rank second.

Answer choice (D) is incorrect because S must rank second.

Answer choice (E) is incorrect because from the last rule G is more productive than T, and therefore T can rank fifth at best.
You do not have the required permissions to view the files attached to this post.

Get the most out of your LSAT Prep Plus subscription.

Analyze and track your performance with our Testing and Analytics Package.